3
$\begingroup$

Hi, I'm trying to find all rational points on the surface of the title, in connection with the Euler Brick (AKA Rational Box) problem.

This surface is equivalent to $ x^2 z^2 - 1 = (x^2 - z^2) y^2 $, and it is easy to find an addition formula for rational points by jogging the second into a form conformable with the equations defining Jacobian elliptic functions:

A rational point on the second form clearly implies rational $a, b, .. e$ (and conversely) satisfying the pair $ (a b)^2 - 1 = e (a b c)^2 $ and $ (a b)^2 - b^4 = e (a b d)^2 $ and dividing each by $ (a b)^2 $, one can express the variables as follows with modulus $ k = b^2 $ :

$ sn(k,u) = 1 / (a b) $

$ cn(k,u) = \sqrt e . c $

$ dn(k,u) = \sqrt e . d $

Because in the addition formulae for Jacobian elliptic functions, $cn$ and $dn$ arise only in products of pairs, and "cross products" $cn . dn$ occur only in the resulting $sn$, this means that a pair of rational solutions with $e = e_1$ and $e = e_2$ implies a rational solution with $E = e_1 e_2$ to the pair:

$ C^2 + E S^2 = 1 $

$ D^2 + b^4 E S^2 = 1 $

Treating this as a Diophantine pair in its own right, without reference to Elliptic functions, one can absorb $ S^2 $ into E, and plug the expression for $E$ given by the first into the second to obtain:

$ D^2 - b^4 C^2 = 1 - b^4 $

For any given $b$ this is a conic curve, with a rational point $ C, D = 1, 1 $ and hence has a rational parametrization over Q, in b^4 and some parameter t say.

Now I'm sure the original surface is not rational (I gather it is a K3 surface). But I wonder if the above can be reversed, to some extent, to at least give a procedure for finding all rational points on the surface from those of the pencil of conics parametrized by $b$.

$\endgroup$
1
  • $\begingroup$ A related paper: J. Top and N. Yui, Congruent number problems and their variants, in: Algorithmic number theory: lattices, number fields, curves and cryptography, 613-–639 (Math. Sci. Res. Inst. Publ. 44, Cambridge Univ. Press, Cambridge, 2008). $\endgroup$ Jan 2, 2011 at 21:42

0

Your Answer

By clicking “Post Your Answer”, you agree to our terms of service and acknowledge you have read our privacy policy.

Browse other questions tagged or ask your own question.